Gandania

This topic has expert replies
Master | Next Rank: 500 Posts
Posts: 228
Joined: Sun Aug 17, 2008 8:08 am
Thanked: 4 times

Gandania

by jainrahul1985 » Sat Jun 25, 2011 11:38 pm
In Gandania, where the government has a monopoly on tobacco sales, the incidence of smoking-related health problems has risen steadily for the last twenty years. The health secretary recently proposed a series of laws aimed at curtailing tobacco use in Gandania. Profits from tobacco sales, however, account for ten percent of Gandania's annual revenues. Therefore, Gandania cannot afford to institute the proposed laws.
Which of the following, if true, most seriously weakens the argument?
A. All health care in Gandania is government-funded.
B. Implementing the proposed laws is not likely to cause a significant increase in the amount of tobacco Gandania exports.
C. The percentage of revenue Gandania receives from tobacco sales has remained steady in recent years.
D. Profits from tobacco sales far surpass any other single source of revenue for the Gandanian government.
E. No government official in Gandania has ever previously proposed laws aimed at curtailing tobacco use

OA A

Experts please suggest why A is correct and B is wrong

Senior | Next Rank: 100 Posts
Posts: 40
Joined: Sun Apr 17, 2011 12:22 pm
Thanked: 5 times
Followed by:8 members
GMAT Score:740

by dubinsky » Sat Jun 25, 2011 11:53 pm
Over here A is the required answer as it implies that the health problems caused by the increase in smoking related health issues will have to be borne by the government.
that is the more the health of citizens decline the country will have to spend on them for their reccovery.......


whereas B does not cause the argument to be weakened in any sort of way.

User avatar
GMAT Instructor
Posts: 905
Joined: Sun Sep 12, 2010 1:38 am
Thanked: 378 times
Followed by:123 members
GMAT Score:760

by Geva@EconomistGMAT » Sun Jun 26, 2011 12:07 am
jainrahul1985 wrote:In Gandania, where the government has a monopoly on tobacco sales, the incidence of smoking-related health problems has risen steadily for the last twenty years. The health secretary recently proposed a series of laws aimed at curtailing tobacco use in Gandania. Profits from tobacco sales, however, account for ten percent of Gandania's annual revenues. Therefore, Gandania cannot afford to institute the proposed laws.
Which of the following, if true, most seriously weakens the argument?
A. All health care in Gandania is government-funded.
B. Implementing the proposed laws is not likely to cause a significant increase in the amount of tobacco Gandania exports.
C. The percentage of revenue Gandania receives from tobacco sales has remained steady in recent years.
D. Profits from tobacco sales far surpass any other single source of revenue for the Gandanian government.
E. No government official in Gandania has ever previously proposed laws aimed at curtailing tobacco use

OA A

Experts please suggest why A is correct and B is wrong
I'm not crazy about A, as it requires too many additional steps to weaken the argument, but B actually does the reverse and strengthens the argument. If the amount of tobacco exports were to somehow increase as a result of these laws, the additional export revenues could compensate for the lost revenues due to reduced internal use, and Gandania could MAYBE afford to implement its laws. Since B says that the export revenues will probably NOT increase, this potential solution to the funding problem is not valid, thus strengthening the argument that Gandania cannot afford to institute the proposed laws.
Geva
Senior Instructor
Master GMAT
1-888-780-GMAT
https://www.mastergmat.com

GMAT/MBA Expert

User avatar
GMAT Instructor
Posts: 3380
Joined: Mon Mar 03, 2008 1:20 am
Thanked: 2256 times
Followed by:1535 members
GMAT Score:800

by lunarpower » Wed Jun 29, 2011 1:34 am
i received a pm.

the conclusion of this argument is that the government cannot afford to fight smoking, because it would not be able to withstand the reduction or elimination of tobacco-related revenues.
the fact that the government revenues would decrease as a result of fighting smoking is just that -- a fact. therefore, that part of the argument can't be weakened.
plus, the only remaining way to weaken this argument would be to show that fighting smoking would also cause a reduction of other government expenses; this is what choice (a) does.
Ron has been teaching various standardized tests for 20 years.

--

Pueden hacerle preguntas a Ron en castellano
Potete chiedere domande a Ron in italiano
On peut poser des questions à Ron en français
Voit esittää kysymyksiä Ron:lle myös suomeksi

--

Quand on se sent bien dans un vêtement, tout peut arriver. Un bon vêtement, c'est un passeport pour le bonheur.

Yves Saint-Laurent

--

Learn more about ron

Junior | Next Rank: 30 Posts
Posts: 18
Joined: Sat Nov 28, 2009 11:21 am
Location: London

by godspeed » Sun Jul 03, 2011 5:30 pm
Ok. IMO - B - Implementing the proposed laws is not likely to cause a significant increase in the amount of tobacco Gandania exports.

Approach -the proposed laws is not likely to cause a significant increase in the export- this means that the local consumption is less. So we could curtail the use of tobacco in the state and still not lose valuable revenue.

Also in A, we don't know how much the government spends on smoking related health problems. And we also don't know how much of the 10 percent revenue comes from export of tobacco and from the local sale of tobacco ?
~GS

GMAT/MBA Expert

User avatar
GMAT Instructor
Posts: 3380
Joined: Mon Mar 03, 2008 1:20 am
Thanked: 2256 times
Followed by:1535 members
GMAT Score:800

by lunarpower » Wed Jul 13, 2011 1:11 pm
godspeed wrote:Ok. IMO - B - Implementing the proposed laws is not likely to cause a significant increase in the amount of tobacco Gandania exports.
Approach -the proposed laws is not likely to cause a significant increase in the export- this means that the local consumption is less. So we could curtail the use of tobacco in the state and still not lose valuable revenue.


i'm not following you here -- this is a statement about exports, and so is unrelated to the magnitude of domestic (local) consumption.

choice (b) actually strengthens the argument, because it assures us that there is no possibility of recouping the lost local revenue by turning to export markets instead.
Also in A, we don't know how much the government spends on smoking related health problems. And we also don't know how much of the 10 percent revenue comes from export of tobacco and from the local sale of tobacco ?
this is a strengthen/weaken problem. therefore, the correct answer choice needs only to STRENGTHEN or WEAKEN the argument (in this case weaken); a strengthener does not have to completely confirm an argument, nor does a weakener have to completely destroy it.
Ron has been teaching various standardized tests for 20 years.

--

Pueden hacerle preguntas a Ron en castellano
Potete chiedere domande a Ron in italiano
On peut poser des questions à Ron en français
Voit esittää kysymyksiä Ron:lle myös suomeksi

--

Quand on se sent bien dans un vêtement, tout peut arriver. Un bon vêtement, c'est un passeport pour le bonheur.

Yves Saint-Laurent

--

Learn more about ron

User avatar
Master | Next Rank: 500 Posts
Posts: 407
Joined: Tue Jan 25, 2011 9:19 am
Thanked: 25 times
Followed by:7 members

by Ozlemg » Thu Jul 14, 2011 11:49 am
i still can not understand why A is correct?

The government cannot afford to institute the law; because of the %100 government paid health care service? This does not weaken the conclusion? -->Gandania cannot afford to institute the proposed laws.

The case is whether the gov. can afford to intstitue the law or can not . The conclusion is not whether to institute the law or not! So shouldnt we weaken the conclusion? Bit confused!
The more you suffer before the test, the less you will do so in the test! :)

GMAT/MBA Expert

User avatar
GMAT Instructor
Posts: 3380
Joined: Mon Mar 03, 2008 1:20 am
Thanked: 2256 times
Followed by:1535 members
GMAT Score:800

by lunarpower » Fri Jul 15, 2011 3:01 am
Ozlemg wrote:i still can not understand why A is correct?

The government cannot afford to institute the law; because of the %100 government paid health care service? This does not weaken the conclusion? -->Gandania cannot afford to institute the proposed laws.

The case is whether the gov. can afford to intstitue the law or can not . The conclusion is not whether to institute the law or not! So shouldnt we weaken the conclusion? Bit confused!
according to (a) the government pays for all health care. therefore, if smoking-related health problems decrease, then the amount of money paid by the government to treat those problems will likewise decrease.
that decrease will offset the loss of tobacco revenues, thus making the change more affordable. (to exactly what extent it will offset that loss we don't quite know, but that's not a problem in weakening problems; you don't have to destroy the argument 100%, just weaken it.)
Ron has been teaching various standardized tests for 20 years.

--

Pueden hacerle preguntas a Ron en castellano
Potete chiedere domande a Ron in italiano
On peut poser des questions à Ron en français
Voit esittää kysymyksiä Ron:lle myös suomeksi

--

Quand on se sent bien dans un vêtement, tout peut arriver. Un bon vêtement, c'est un passeport pour le bonheur.

Yves Saint-Laurent

--

Learn more about ron

Legendary Member
Posts: 1404
Joined: Tue May 20, 2008 6:55 pm
Thanked: 18 times
Followed by:2 members

by tanviet » Fri Jul 15, 2011 8:23 pm
lunarpower wrote:i received a pm.

the conclusion of this argument is that the government cannot afford to fight smoking, because it would not be able to withstand the reduction or elimination of tobacco-related revenues.
the fact that the government revenues would decrease as a result of fighting smoking is just that -- a fact. therefore, that part of the argument can't be weakened.
plus, the only remaining way to weaken this argument would be to show that fighting smoking would also cause a reduction of other government expenses; this is what choice (a) does.
Thank you Ron
Please, tell me What is the assumption of the choice A. The weaker must increase doubt of an Assumption. I see that the assumption here is hard to realize. The assumption is revenue reduction can not improve health. The assumption is not that revenue reduction must be avoided-This assumption jump out to us immediately but is fake. I think this question is hard.

GMAT/MBA Expert

User avatar
GMAT Instructor
Posts: 3380
Joined: Mon Mar 03, 2008 1:20 am
Thanked: 2256 times
Followed by:1535 members
GMAT Score:800

by lunarpower » Mon Jul 18, 2011 1:41 am
duongthang wrote:Please, tell me What is the assumption of the choice A. The weaker must increase doubt of an Assumption. I see that the assumption here is hard to realize.
this is not a good way to think about weakening problems in general; it will convert many easy problems into insanely hard ones.

for instance:

Argument:
James is the tallest boy at his school. Therefore, James should try out for the basketball team.

Choice (A):
James has a leg deformity that prevents him from running.

here, it should be extremely obvious that choice (A) weakens the argument.
on the other hand, if you proceed using this "doubt the assumption" tactic, then you would actually have to come up with this assumption -- James doesn't have any deformities that affect basketball -- while reading the argument! that is, for all practical purposes, impossible, yet it's easy to see by direct investigation that this statement is a weakener.

DO NOT APPROACH STRENGTHEN/WEAKEN PROBLEMS WITH RULES!
* learn how the argument works
* for each choice, use common sense, combined with your understanding of the argument,
to tell whether the argument is strengthened or weakened (or neither).

it is IMPOSSIBLE to solve strengthen/weaken problems with memorized rules. if you try, you will actually get worse at them than before you started studying.
Ron has been teaching various standardized tests for 20 years.

--

Pueden hacerle preguntas a Ron en castellano
Potete chiedere domande a Ron in italiano
On peut poser des questions à Ron en français
Voit esittää kysymyksiä Ron:lle myös suomeksi

--

Quand on se sent bien dans un vêtement, tout peut arriver. Un bon vêtement, c'est un passeport pour le bonheur.

Yves Saint-Laurent

--

Learn more about ron

Legendary Member
Posts: 1404
Joined: Tue May 20, 2008 6:55 pm
Thanked: 18 times
Followed by:2 members

by tanviet » Mon Jul 18, 2011 6:48 am
lunarpower wrote:
duongthang wrote:Please, tell me What is the assumption of the choice A. The weaker must increase doubt of an Assumption. I see that the assumption here is hard to realize.
this is not a good way to think about weakening problems in general; it will convert many easy problems into insanely hard ones.

for instance:

Argument:
James is the tallest boy at his school. Therefore, James should try out for the basketball team.

Choice (A):
James has a leg deformity that prevents him from running.

here, it should be extremely obvious that choice (A) weakens the argument.
on the other hand, if you proceed using this "doubt the assumption" tactic, then you would actually have to come up with this assumption -- James doesn't have any deformities that affect basketball -- while reading the argument! that is, for all practical purposes, impossible, yet it's easy to see by direct investigation that this statement is a weakener.

DO NOT APPROACH STRENGTHEN/WEAKEN PROBLEMS WITH RULES!
* learn how the argument works
* for each choice, use common sense, combined with your understanding of the argument,
to tell whether the argument is strengthened or weakened (or neither).

it is IMPOSSIBLE to solve strengthen/weaken problems with memorized rules. if you try, you will actually get worse at them than before you started studying.
Thank you, Ron
your example is very nice. But I feel very effective when I apply the rule that a weakener increases doubt of an assumption. This rule is mentioned in the Kaplan Premier book, a very basic book. I appreciate the book hightly. I see the rule works both for easy and hard problem. the rule is very effective for hard problems which appear on GMATPREP. I like your above example above very much. it is the way GMAT creates hard problem. Esay-to-realize assumption is not relevant but hard-to-realize assumption is relevant. The assumption based questions would be made harder by making the questions based on hard-to-see assumption.

One thing more, Ron. I do not understand the last question of the Critical Reasoning Section of the Princeton Book. The question "video rental" of GMATPREP has similar dificulty. both of these questions are discussed many time in this forum and gmatclub forum. However, even experts do not discuss them properly and consider them wrong. But GMATPREP questions is never wrong but a monster which we do not realize when we do the problems again. I will to email you for this matter.

User avatar
Master | Next Rank: 500 Posts
Posts: 461
Joined: Tue May 10, 2011 9:09 am
Location: pune
Thanked: 36 times
Followed by:3 members

by amit2k9 » Mon Jul 18, 2011 7:25 am
using POE, B,D and E are out.

Only A and C remain.C in fact strengthens the conclusion.

A is a winner here.
For Understanding Sustainability,Green Businesses and Social Entrepreneurship visit -https://aamthoughts.blocked/
(Featured Best Green Site Worldwide-https://bloggers.com/green/popular/page2)